1234567891011121314151617181920212223242526272829303132333435363738394041424344454647484950515253545556575859606162636465 |
- \documentclass[11pt]{article}
- \usepackage{amsthm,amsmath,amsfonts,amssymb}
- \usepackage[ruled,noline,noend]{algorithm2e}
- \usepackage[margin=1in]{geometry}
- \usepackage{enumerate}
- %%% Update the following commands with your name, ID, and acknowledgments.
- \newcommand{\YourName}{Tareef Dedhar}
- \newcommand{\YourStudentID}{20621325}
- \newcommand{\YourAck}{This question was completed only with the teachings from CS courses here at uWaterloo.}
- \newenvironment{solution}{\paragraph{Solution.}}{}
- \title{
- \vspace{-0.8in} \normalsize
- \begin{flushright} \YourName \\ \YourStudentID \end{flushright}
- \rule{\textwidth}{.5pt}\\[0.4cm] \huge Assignment 1: Question 1 \\
- \rule{\textwidth}{2pt}
- \vspace{-1in}
- }
- \date{}
- \begin{document}
- \maketitle
- \paragraph{Acknowledgments.} \YourAck
- %%%%%%%%%%%%%%%%%%%%%%%%%%%%%%%%%%%%
- \section*{Asymptotics}
- Prove or disprove each of the following statements.
- \begin{enumerate}[(a)]
- \item For any constant $b > 0$, the function $f : n \mapsto 1 + b + b^2 + b^3 + \cdots + b^n$ satisfies
- \[
- f(n) = \begin{cases}
- \Theta(b^n) & \mbox{if } b > 1 \\
- \Theta(1) & \mbox{if } b \le 1.
- \end{cases}
- \]
- \begin{solution}
- Since f(n) is a geometric series, we can express it as $\sum_{i=1}^{n} {b^i}$ = $\frac{1 - {b^N}}{1 - b}$ = $\frac{{b^n} - 1}{b - 1}$
- Using this:\\
- When b=1: $\lim{n\to\infty}{\dfrac{\tfrac{{b^n} - 1}{b - 1}}{b^n}}$=$\frac{{b^n} - 1}{{b^{n+1}} - b}$, which, by :'Hopital's rule = $1 \div b$. Since $0 \lt 1 \div b \lt \infty$ , we have a \theta bound.
- \end{solution}
- \item For every pair of functions $f, g : \mathbb{Z}^+ \to \mathbb{R}^{\ge 1}$ that satisfy $f = \Theta(g)$, the functions $F : n \mapsto 2^{f(n)}$ and $G : n \mapsto 2^{g(n)}$ also satisfy $F = \Theta(G)$.
- \begin{solution}
- (ENTER YOUR SOLUTION HERE.)
- \end{solution}
- \item For every pair of functions $f, g : \mathbb{Z}^+ \to \mathbb{R}^{\ge 1}$ that satisfy $f = o(g)$, the functions $F : n \mapsto 2^{f(n)}$ and $G : n \mapsto 2^{g(n)}$ also satisfy $F = o(G)$.
- \begin{solution}
- (ENTER YOUR SOLUTION HERE.)
- \end{solution}
- \end{enumerate}
- \end{document}
|